Very Hard System of Equations












-1












$begingroup$


Solve the system of equations:



begin{cases}
sqrt{xy}(x + 3y)(3x + y) = 14 \
(x + y)(x^2 + y^2 + 14xy) = 36.
end{cases}



Suppose $x + y = m$ and $xy = n$. So I get



begin{cases}
(3m^2+4n) sqrt n = 14 \
m^3+12mn=36
end{cases}



So can express $m$ in terms of $n$ but I want only real solution.
Is there a faster way?










share|cite|improve this question











$endgroup$








  • 1




    $begingroup$
    It will help you greatly to note that these equations are transposable, that is, $(x,y)=(X,Y)$ is a solution, so too is $(x,y)=(Y,X)$.
    $endgroup$
    – Rhys Hughes
    Jan 18 at 3:22
















-1












$begingroup$


Solve the system of equations:



begin{cases}
sqrt{xy}(x + 3y)(3x + y) = 14 \
(x + y)(x^2 + y^2 + 14xy) = 36.
end{cases}



Suppose $x + y = m$ and $xy = n$. So I get



begin{cases}
(3m^2+4n) sqrt n = 14 \
m^3+12mn=36
end{cases}



So can express $m$ in terms of $n$ but I want only real solution.
Is there a faster way?










share|cite|improve this question











$endgroup$








  • 1




    $begingroup$
    It will help you greatly to note that these equations are transposable, that is, $(x,y)=(X,Y)$ is a solution, so too is $(x,y)=(Y,X)$.
    $endgroup$
    – Rhys Hughes
    Jan 18 at 3:22














-1












-1








-1





$begingroup$


Solve the system of equations:



begin{cases}
sqrt{xy}(x + 3y)(3x + y) = 14 \
(x + y)(x^2 + y^2 + 14xy) = 36.
end{cases}



Suppose $x + y = m$ and $xy = n$. So I get



begin{cases}
(3m^2+4n) sqrt n = 14 \
m^3+12mn=36
end{cases}



So can express $m$ in terms of $n$ but I want only real solution.
Is there a faster way?










share|cite|improve this question











$endgroup$




Solve the system of equations:



begin{cases}
sqrt{xy}(x + 3y)(3x + y) = 14 \
(x + y)(x^2 + y^2 + 14xy) = 36.
end{cases}



Suppose $x + y = m$ and $xy = n$. So I get



begin{cases}
(3m^2+4n) sqrt n = 14 \
m^3+12mn=36
end{cases}



So can express $m$ in terms of $n$ but I want only real solution.
Is there a faster way?







algebra-precalculus systems-of-equations substitution






share|cite|improve this question















share|cite|improve this question













share|cite|improve this question




share|cite|improve this question








edited Jan 18 at 12:41









Michael Rozenberg

103k1891195




103k1891195










asked Jan 18 at 2:44









HeartHeart

28918




28918








  • 1




    $begingroup$
    It will help you greatly to note that these equations are transposable, that is, $(x,y)=(X,Y)$ is a solution, so too is $(x,y)=(Y,X)$.
    $endgroup$
    – Rhys Hughes
    Jan 18 at 3:22














  • 1




    $begingroup$
    It will help you greatly to note that these equations are transposable, that is, $(x,y)=(X,Y)$ is a solution, so too is $(x,y)=(Y,X)$.
    $endgroup$
    – Rhys Hughes
    Jan 18 at 3:22








1




1




$begingroup$
It will help you greatly to note that these equations are transposable, that is, $(x,y)=(X,Y)$ is a solution, so too is $(x,y)=(Y,X)$.
$endgroup$
– Rhys Hughes
Jan 18 at 3:22




$begingroup$
It will help you greatly to note that these equations are transposable, that is, $(x,y)=(X,Y)$ is a solution, so too is $(x,y)=(Y,X)$.
$endgroup$
– Rhys Hughes
Jan 18 at 3:22










2 Answers
2






active

oldest

votes


















0












$begingroup$

Let $x+y=2ksqrt{xy}.$
Thus, by AM-GM $kgeq1$ and from the first and the second equations we obtain:
$$18sqrt{xy}(3x^2+3y^2+10xy)=7(x+y)(x^2+y^2+14xy)$$ or
$$18(12k^2+4)=7cdot2k(4k^2+12)$$ or
$$7k^3-27k^2+21k-9=0$$ or
$$7k^3-21k^2-6k^2+18k+3k-9=0$$ or
$$(k-3)(7k^2-6k+3)=0,$$ which gives $k=3$ and $$x+y=6sqrt{xy}.$$
Can you end it now?






share|cite|improve this answer









$endgroup$













  • $begingroup$
    Yes thank you for good solution :)
    $endgroup$
    – Heart
    Jan 18 at 14:34










  • $begingroup$
    @Heart You are welcome!
    $endgroup$
    – Michael Rozenberg
    Jan 18 at 17:02



















2












$begingroup$

Solving the second equation for $n$ we get $$n=-1/12,{frac {{m}^{3}-36}{m}}$$ for $$mneq 0$$ plugging this in your first equation we get
$$1/6, left( 3,{m}^{2}-1/3,{frac {{m}^{3}-36}{m}} right) sqrt {-3
,{frac {{m}^{3}-36}{m}}}=14
$$

so now we square and factorize this equation we obtain:
$$-16, left( m-3 right) left( {m}^{2}+3 right) left( {m}^{2}+3,
m+3 right) left( {m}^{2}-3,m+3 right) left( {m}^{2}+3,m+9
right)
=0$$

I hope you will find all solutions.






share|cite|improve this answer









$endgroup$













    Your Answer





    StackExchange.ifUsing("editor", function () {
    return StackExchange.using("mathjaxEditing", function () {
    StackExchange.MarkdownEditor.creationCallbacks.add(function (editor, postfix) {
    StackExchange.mathjaxEditing.prepareWmdForMathJax(editor, postfix, [["$", "$"], ["\\(","\\)"]]);
    });
    });
    }, "mathjax-editing");

    StackExchange.ready(function() {
    var channelOptions = {
    tags: "".split(" "),
    id: "69"
    };
    initTagRenderer("".split(" "), "".split(" "), channelOptions);

    StackExchange.using("externalEditor", function() {
    // Have to fire editor after snippets, if snippets enabled
    if (StackExchange.settings.snippets.snippetsEnabled) {
    StackExchange.using("snippets", function() {
    createEditor();
    });
    }
    else {
    createEditor();
    }
    });

    function createEditor() {
    StackExchange.prepareEditor({
    heartbeatType: 'answer',
    autoActivateHeartbeat: false,
    convertImagesToLinks: true,
    noModals: true,
    showLowRepImageUploadWarning: true,
    reputationToPostImages: 10,
    bindNavPrevention: true,
    postfix: "",
    imageUploader: {
    brandingHtml: "Powered by u003ca class="icon-imgur-white" href="https://imgur.com/"u003eu003c/au003e",
    contentPolicyHtml: "User contributions licensed under u003ca href="https://creativecommons.org/licenses/by-sa/3.0/"u003ecc by-sa 3.0 with attribution requiredu003c/au003e u003ca href="https://stackoverflow.com/legal/content-policy"u003e(content policy)u003c/au003e",
    allowUrls: true
    },
    noCode: true, onDemand: true,
    discardSelector: ".discard-answer"
    ,immediatelyShowMarkdownHelp:true
    });


    }
    });














    draft saved

    draft discarded


















    StackExchange.ready(
    function () {
    StackExchange.openid.initPostLogin('.new-post-login', 'https%3a%2f%2fmath.stackexchange.com%2fquestions%2f3077771%2fvery-hard-system-of-equations%23new-answer', 'question_page');
    }
    );

    Post as a guest















    Required, but never shown

























    2 Answers
    2






    active

    oldest

    votes








    2 Answers
    2






    active

    oldest

    votes









    active

    oldest

    votes






    active

    oldest

    votes









    0












    $begingroup$

    Let $x+y=2ksqrt{xy}.$
    Thus, by AM-GM $kgeq1$ and from the first and the second equations we obtain:
    $$18sqrt{xy}(3x^2+3y^2+10xy)=7(x+y)(x^2+y^2+14xy)$$ or
    $$18(12k^2+4)=7cdot2k(4k^2+12)$$ or
    $$7k^3-27k^2+21k-9=0$$ or
    $$7k^3-21k^2-6k^2+18k+3k-9=0$$ or
    $$(k-3)(7k^2-6k+3)=0,$$ which gives $k=3$ and $$x+y=6sqrt{xy}.$$
    Can you end it now?






    share|cite|improve this answer









    $endgroup$













    • $begingroup$
      Yes thank you for good solution :)
      $endgroup$
      – Heart
      Jan 18 at 14:34










    • $begingroup$
      @Heart You are welcome!
      $endgroup$
      – Michael Rozenberg
      Jan 18 at 17:02
















    0












    $begingroup$

    Let $x+y=2ksqrt{xy}.$
    Thus, by AM-GM $kgeq1$ and from the first and the second equations we obtain:
    $$18sqrt{xy}(3x^2+3y^2+10xy)=7(x+y)(x^2+y^2+14xy)$$ or
    $$18(12k^2+4)=7cdot2k(4k^2+12)$$ or
    $$7k^3-27k^2+21k-9=0$$ or
    $$7k^3-21k^2-6k^2+18k+3k-9=0$$ or
    $$(k-3)(7k^2-6k+3)=0,$$ which gives $k=3$ and $$x+y=6sqrt{xy}.$$
    Can you end it now?






    share|cite|improve this answer









    $endgroup$













    • $begingroup$
      Yes thank you for good solution :)
      $endgroup$
      – Heart
      Jan 18 at 14:34










    • $begingroup$
      @Heart You are welcome!
      $endgroup$
      – Michael Rozenberg
      Jan 18 at 17:02














    0












    0








    0





    $begingroup$

    Let $x+y=2ksqrt{xy}.$
    Thus, by AM-GM $kgeq1$ and from the first and the second equations we obtain:
    $$18sqrt{xy}(3x^2+3y^2+10xy)=7(x+y)(x^2+y^2+14xy)$$ or
    $$18(12k^2+4)=7cdot2k(4k^2+12)$$ or
    $$7k^3-27k^2+21k-9=0$$ or
    $$7k^3-21k^2-6k^2+18k+3k-9=0$$ or
    $$(k-3)(7k^2-6k+3)=0,$$ which gives $k=3$ and $$x+y=6sqrt{xy}.$$
    Can you end it now?






    share|cite|improve this answer









    $endgroup$



    Let $x+y=2ksqrt{xy}.$
    Thus, by AM-GM $kgeq1$ and from the first and the second equations we obtain:
    $$18sqrt{xy}(3x^2+3y^2+10xy)=7(x+y)(x^2+y^2+14xy)$$ or
    $$18(12k^2+4)=7cdot2k(4k^2+12)$$ or
    $$7k^3-27k^2+21k-9=0$$ or
    $$7k^3-21k^2-6k^2+18k+3k-9=0$$ or
    $$(k-3)(7k^2-6k+3)=0,$$ which gives $k=3$ and $$x+y=6sqrt{xy}.$$
    Can you end it now?







    share|cite|improve this answer












    share|cite|improve this answer



    share|cite|improve this answer










    answered Jan 18 at 7:06









    Michael RozenbergMichael Rozenberg

    103k1891195




    103k1891195












    • $begingroup$
      Yes thank you for good solution :)
      $endgroup$
      – Heart
      Jan 18 at 14:34










    • $begingroup$
      @Heart You are welcome!
      $endgroup$
      – Michael Rozenberg
      Jan 18 at 17:02


















    • $begingroup$
      Yes thank you for good solution :)
      $endgroup$
      – Heart
      Jan 18 at 14:34










    • $begingroup$
      @Heart You are welcome!
      $endgroup$
      – Michael Rozenberg
      Jan 18 at 17:02
















    $begingroup$
    Yes thank you for good solution :)
    $endgroup$
    – Heart
    Jan 18 at 14:34




    $begingroup$
    Yes thank you for good solution :)
    $endgroup$
    – Heart
    Jan 18 at 14:34












    $begingroup$
    @Heart You are welcome!
    $endgroup$
    – Michael Rozenberg
    Jan 18 at 17:02




    $begingroup$
    @Heart You are welcome!
    $endgroup$
    – Michael Rozenberg
    Jan 18 at 17:02











    2












    $begingroup$

    Solving the second equation for $n$ we get $$n=-1/12,{frac {{m}^{3}-36}{m}}$$ for $$mneq 0$$ plugging this in your first equation we get
    $$1/6, left( 3,{m}^{2}-1/3,{frac {{m}^{3}-36}{m}} right) sqrt {-3
    ,{frac {{m}^{3}-36}{m}}}=14
    $$

    so now we square and factorize this equation we obtain:
    $$-16, left( m-3 right) left( {m}^{2}+3 right) left( {m}^{2}+3,
    m+3 right) left( {m}^{2}-3,m+3 right) left( {m}^{2}+3,m+9
    right)
    =0$$

    I hope you will find all solutions.






    share|cite|improve this answer









    $endgroup$


















      2












      $begingroup$

      Solving the second equation for $n$ we get $$n=-1/12,{frac {{m}^{3}-36}{m}}$$ for $$mneq 0$$ plugging this in your first equation we get
      $$1/6, left( 3,{m}^{2}-1/3,{frac {{m}^{3}-36}{m}} right) sqrt {-3
      ,{frac {{m}^{3}-36}{m}}}=14
      $$

      so now we square and factorize this equation we obtain:
      $$-16, left( m-3 right) left( {m}^{2}+3 right) left( {m}^{2}+3,
      m+3 right) left( {m}^{2}-3,m+3 right) left( {m}^{2}+3,m+9
      right)
      =0$$

      I hope you will find all solutions.






      share|cite|improve this answer









      $endgroup$
















        2












        2








        2





        $begingroup$

        Solving the second equation for $n$ we get $$n=-1/12,{frac {{m}^{3}-36}{m}}$$ for $$mneq 0$$ plugging this in your first equation we get
        $$1/6, left( 3,{m}^{2}-1/3,{frac {{m}^{3}-36}{m}} right) sqrt {-3
        ,{frac {{m}^{3}-36}{m}}}=14
        $$

        so now we square and factorize this equation we obtain:
        $$-16, left( m-3 right) left( {m}^{2}+3 right) left( {m}^{2}+3,
        m+3 right) left( {m}^{2}-3,m+3 right) left( {m}^{2}+3,m+9
        right)
        =0$$

        I hope you will find all solutions.






        share|cite|improve this answer









        $endgroup$



        Solving the second equation for $n$ we get $$n=-1/12,{frac {{m}^{3}-36}{m}}$$ for $$mneq 0$$ plugging this in your first equation we get
        $$1/6, left( 3,{m}^{2}-1/3,{frac {{m}^{3}-36}{m}} right) sqrt {-3
        ,{frac {{m}^{3}-36}{m}}}=14
        $$

        so now we square and factorize this equation we obtain:
        $$-16, left( m-3 right) left( {m}^{2}+3 right) left( {m}^{2}+3,
        m+3 right) left( {m}^{2}-3,m+3 right) left( {m}^{2}+3,m+9
        right)
        =0$$

        I hope you will find all solutions.







        share|cite|improve this answer












        share|cite|improve this answer



        share|cite|improve this answer










        answered Jan 18 at 3:23









        Dr. Sonnhard GraubnerDr. Sonnhard Graubner

        75.2k42865




        75.2k42865






























            draft saved

            draft discarded




















































            Thanks for contributing an answer to Mathematics Stack Exchange!


            • Please be sure to answer the question. Provide details and share your research!

            But avoid



            • Asking for help, clarification, or responding to other answers.

            • Making statements based on opinion; back them up with references or personal experience.


            Use MathJax to format equations. MathJax reference.


            To learn more, see our tips on writing great answers.




            draft saved


            draft discarded














            StackExchange.ready(
            function () {
            StackExchange.openid.initPostLogin('.new-post-login', 'https%3a%2f%2fmath.stackexchange.com%2fquestions%2f3077771%2fvery-hard-system-of-equations%23new-answer', 'question_page');
            }
            );

            Post as a guest















            Required, but never shown





















































            Required, but never shown














            Required, but never shown












            Required, but never shown







            Required, but never shown

































            Required, but never shown














            Required, but never shown












            Required, but never shown







            Required, but never shown







            Popular posts from this blog

            Mario Kart Wii

            What does “Dominus providebit” mean?

            Antonio Litta Visconti Arese